Pytania otagowane jako bounds


2
Jak możemy ograniczyć prawdopodobieństwo, że zmienna losowa jest maksymalna?
\newcommand{\P}{\mathbb{P}} Załóżmy, że mamy NNN niezależnych zmiennych losowych X1X1X_1 , ……\ldots , XnXnX_n ze skończonymi środkami μ1≤…≤μNμ1≤…≤μN\mu_1 \leq \ldots \leq \mu_N i wariancji σ21σ12\sigma_1^2 , ……\ldots , σ2NσN2\sigma_N^2 . Szukam granic bez dystrybucji prawdopodobieństwa, że ​​każdy Xi≠XNXi≠XNX_i \neq X_N jest większy niż wszystkie inne XjXjX_j , j≠ij≠ij \neq i . …



1
Górne granice gęstości kopuły?
Fréchet-Hoeffding górna granica odnosi się do funkcji rozkładu kopułą i jest przekazywana przez C(u1,...,ud)≤min{u1,..,ud}.C(u1,...,ud)≤min{u1,..,ud}.C(u_1,...,u_d)\leq \min\{u_1,..,u_d\}. Czy istnieje podobna (w tym sensie, że zależy to od gęstości krańcowej) górna granica gęstości kopuły c(u1,...,ud)c(u1,...,ud)c(u_1,...,u_d) zamiast CDF? Wszelkie odniesienia będą mile widziane.


2
Jaka jest wariancja maksimum próbki?
Szukam granic wariancji maksimum zestawu zmiennych losowych. Innymi słowy, szukam formuł zamkniętych dla bBB , takich że Var ( maksjaXja) ≤ B,Var(maxiXi)≤B, \mbox{Var}(\max_i X_i) \leq B \enspace, Gdzie X= { X1, … , XM.}X={X1,…,XM}X = \{ X_1, \ldots, X_M \} jest stałym zestawem M.MM zmiennych losowych o skończonych środkach μ1, …



1
Radzenie sobie z regresją niezwykle ograniczonej zmiennej odpowiedzi
Próbuję modelować zmienną odpowiedzi, która teoretycznie jest ograniczona między -225 a +225. Zmienna to łączny wynik uzyskany przez badanych podczas gry. Chociaż teoretycznie możliwe jest zdobycie przez uczestników +225 punktów. Pomimo tego, ponieważ wynik zależał nie tylko od działań podmiotów, ale także działań innych działań, maksymalna liczba zdobytych punktów wyniosła …

2
Testowanie hipotez i całkowity dystans wariancji vs. dywergencja Kullbacka-Leiblera
W moich badaniach natrafiłem na następujący ogólny problem: mam dwie rozkłady i w tej samej domenie i dużą (ale skończoną) liczbę próbek z tych rozkładów. Próbki są niezależnie i identycznie rozmieszczone z jednego z tych dwóch rozkładów (chociaż rozkłady mogą być powiązane: na przykład Q może być mieszaniną P i …

1
Ogranicza różnicę skorelowanych zmiennych losowych
Biorąc pod uwagę dwie wysoce skorelowane zmienne losowe XXX i YYY, Chciałbym ograniczyć prawdopodobieństwo różnicy |X−Y||X−Y| |X - Y| przekracza pewną kwotę: P(|X−Y|&gt;K)&lt;δP(|X−Y|&gt;K)&lt;δ P( |X - Y| > K) < \delta Załóż dla uproszczenia, że: Współczynnik korelacji jest znany jako „wysoki”, powiedzmy: ρX,Y=covar(X,Y)/σXσY≥1−ϵρX,Y=covar(X,Y)/σXσY≥1−ϵ \rho_{X,Y}= {covar(X,Y)} / {\sigma_X \sigma_Y} \geq 1 …

3
Jak to udowodnić
Próbowałem ustalić nierówność |Ti|=∣∣Xi−X¯∣∣S≤n−1n−−√|Ti|=|Xi−X¯|S≤n−1n\left| T_i \right|=\frac{\left|X_i -\bar{X} \right|}{S} \leq\frac{n-1}{\sqrt{n}} gdzie to średnia próbki, a standardowe odchylenie próbki, to znaczy .X¯X¯\bar{X}SSSS=∑ni=1(Xi−X¯)2n−1−−−−−−−−−√S=∑i=1n(Xi−X¯)2n−1S=\sqrt{\frac{\sum_{i=1}^n \left( X_i -\bar{X} \right)^2}{n-1}} Łatwo zauważyć, że a więc ale to nie jest bardzo blisko tego, czego szukałem, ani nie jest to przydatne ograniczenie. Eksperymentowałem z Cauchy-Schwarzem i nierównościami trójkąta, …

2
Wyniki regresji mają nieoczekiwany górny limit
Próbuję przewidzieć wynik równowagi i wypróbowałem kilka różnych metod regresji. Zauważyłem jedną rzecz, że przewidywane wartości wydają się mieć pewien górny limit. To znaczy, faktyczny bilans wynosi , ale moje przewidywania sięgają około . Poniższy wykres pokazuje rzeczywistą vs przewidywaną równowagę (przewidywaną za pomocą regresji liniowej):[ 0,0 , 1,0 )[0,0,1.0)[0.0, …
Korzystając z naszej strony potwierdzasz, że przeczytałeś(-aś) i rozumiesz nasze zasady używania plików cookie i zasady ochrony prywatności.
Licensed under cc by-sa 3.0 with attribution required.